\( \newcommand{\nPr}[2]{{}^{#1}A_{#2} } \newcommand{\combin}[2]{{}^{#1}C_{#2} } \newcommand{\cmod}[3]{#1 \equiv #2\left(\bmod {}{#3}\right)} \newcommand{\frc}[2]{\displaystyle\frac{#1}{#2}} \newcommand{\mdc}[2]{\left( {#1},{#2}\right)} \newcommand{\mmc}[2]{\left[ {#1},{#2}\right]} \newcommand{\cis}{\mathop{\rm cis}} \newcommand{\ImP}{\mathop{\rm Im}} \newcommand{\ReP}{\mathop{\rm Re}} \newcommand{\sen}{\mathop{\rm sen}} \newcommand{\tg}{\mathop{\rm tg}} \newcommand{\cotg}{\mathop{\rm cotg}} \newcommand{\cosec}{\mathop{\rm cosec}} \newcommand{\cotgh}{\mathop{\rm cotgh}} \newcommand{\cosech}{\mathop{\rm cosech}} \newcommand{\sech}{\mathop{\rm sech}} \newcommand{\sh}{\mathop{\rm sh}} \newcommand{\ch}{\mathop{\rm ch}} \newcommand{\th}{\mathop{\rm th}} \newcommand{\senEL}[1]{\mathop{\rm sen}^{#1}} \newcommand{\tgEL}[1]{\mathop{\rm tg}^{#1}} \newcommand{\cotgEL}[1]{\mathop{\rm cotg}^{#1}} \newcommand{\cosecEL}{\mathop{\rm cosec}^{#1}} \newcommand{\shEL}[1]{\mathop{\rm sh^{#1}}} \newcommand{\chEL}[1]{\mathop{\rm ch^{#1}}} \newcommand{\thEL}[1]{\mathop{\rm th^{#1}}} \newcommand{\cotghEL}[1]{\mathop{\rm cotgh^{#1}}} \newcommand{\cosechEL}[1]{\mathop{\rm cosech^{#1}}} \newcommand{\sechEL}[1]{\mathop{\rm sech^{#1}}} \newcommand{\senq}{\senEL{2}} \newcommand{\tgq}{\tgEL{2}} \newcommand{\cotgq}{\cotgEL{2}} \newcommand{\cosecq}{\cosecEL{2}} \newcommand{\cotghq}{\cotghEL{2}} \newcommand{\cosechq}{\cosechEL{2}} \newcommand{\sechq}{\sechEL{2}} \newcommand{\shq}{\shEL{2}} \newcommand{\chq}{\chEL{2}} \newcommand{\arctg}{\mathop{\rm arctg}} \newcommand{\arcsen}{\mathop{\rm arcsen}} \newcommand{\argsh}{\mathop{\rm argsh}} \newcommand{\argch}{\mathop{\rm argch}} \newcommand{\Var}{\mathop{\rm Var}} \newcommand{\vect}[1]{\overrightarrow{#1}} \newcommand{\tr}[1]{ \textnormal{Tr}\left({#1}\right)} \newcommand{\C}{\mathbb{C}} \newcommand{\E}{\mathbb{E}} \newcommand{\H}{\mathbb{H}} \newcommand{\I}{\mathbb{I}} \newcommand{\N}{\mathbb{N}} \newcommand{\P}{\mathbb{P}} \newcommand{\Q}{\mathbb{Q}} \newcommand{\R}{\mathbb{R}} \newcommand{\Z}{\mathbb{Z}} \newcommand{\til}{\sim} \newcommand{\mdc}{\mathop{\rm m.d.c.}} \newcommand{\mmc}{\mathop{\rm m.m.c.}} \newcommand{\vect}[1]{\overrightarrow{#1}} \newcommand{\dfrc}{\displaystyle\frac} \newcommand{\Mod}[1]{\ (\mathrm{mod}\ #1)} \)

25/12/2019

O seno de 18º

Esta ocorreu-me ao olhar para uma estrela de 5 pontas, neste Natal.
Há uns bons anos, partindo de "uma estrela regular de 5 pontas" (não vamos discutir a precisão matemática desta designação, ok?), ocorreu-me uma forma de deduzir o cosseno de $36^{\circ}$. Está em https://oldcpaulof2.blogspot.com/2013/06/o-numero-de-ouro-parte-3-o-pentagrama-e.html.
Nesse post, eu mostrei que \[\cos 36^{\circ}=\frac{\phi}{2}\] onde $\phi$ é o número de ouro \[\phi=\frac{1+\sqrt{5}}{2}\] Ora, uma fórmula que acabo por utilizar sempre que me aparecem alunos a pedir explicações de cadeiras que envolvem cálculo integral é
\[{\sen}^{2} \alpha=\frac{1-\cos (2\alpha)}{2}\] (um dia destes anexo uns formulários de trigonometria e de séries ao blog...)
Com umas pequenas manipulações algébricas escreve-se \begin{eqnarray*} {{\sen}^{2} 18^{\circ}}&=&{\frac{1-\cos 36^{\circ}}{2}}\\ {}&=&{\frac{1-\displaystyle\frac{1+\sqrt{5}}{4}}{2}}\\ {}&=&{\frac{4-1-\sqrt{5}}{8}}\\ {}&=&{\frac{3-\sqrt{5}}{8}} \end{eqnarray*} Vou tentar transformar $\displaystyle\frac{3-\sqrt{5}}{8}$ no quadrado de um número positivo. \begin{eqnarray*} {\frac{3-\sqrt{5}}{8}}&=&{\frac{6-2\sqrt{5}}{16}}\\ {}&=&{\frac{5-2\sqrt{5}+1}{16}}\\ {}&=&{\frac{\sqrt{5}^2-2\sqrt{5}+1^2}{4^4}}\\ {}&=&{\left(\frac{\sqrt{5}-1}{4}\right)^2} \end{eqnarray*} Conclusão: \[\sen 18^{\circ}=\frac{\sqrt{5}-1}{4}\]
$\blacksquare$
Feliz Natal
Podem ver uma demonstração alternativa, e que se pode apresentar a alunos do ensino secundário em: https://www.youtube.com/watch?v=_00oskWLtII.
Hoje em dia encontra-se de tudo no youtube... mas eu pertenço à velha escola: Prefiro pensar e fazer eu...
Curiosidade diabólica(26/12/2019) \begin{eqnarray*} {\sen 666^{\circ}}&=&{\sen 306^{\circ}}\\ {}&=&{\sen -54^{\circ}}\\ {}&=&{-\sen 54^{\circ}}\\ {}&=&{-\cos 36 ^{\circ}}\\ {}&=&{-\frac{\phi}{2}} \end{eqnarray*}

09/12/2019

Uma curiosidade sobre a constante de Euler-Mascheroni

Em explicações, às vezes aparecem-nos coisas que desconhecíamos, ou que não tínhamos notado até esse momento.
Há uns anos numa lista de exercícios de Probabilidades e Estatística de um aluno da professora Sandra Mendonça (Universidade da Madeira), apareceu, (como curiosidade) a igualdade
\[\gamma=-\Gamma'(1)\]
Onde $\gamma$ é a constante de Euler-Mascheroni.
Fui à minha calculadora, e observei (numericamente) o resultado.
O que se passava é que a definição que eu conhecia de $\gamma$, não era aquela, portanto devia ser possível deduzir a partir da definição que eu tinha, ou de alguma das fórmulas que eu conhecia.
Fui à Wikipedia. Reencontrei a propriedade, mas nada de prova...
Pensei um pouco e consegui chegar à demonstração que deixo aqui hoje.

Vou começar por partilhar convosco um pequeno e antigo pdf meu, de 12 páginas sobre a função Gama (de Euler).

https://drive.google.com/open?id=1Pn2yjn4z0AoLbqozwJcyGfVcdr-p32b4

Nesse pdf, na página 10 recordo a "minha" definição da constante de Euler-Mascheroni :

\[\gamma  = \lim\limits_{n\to \infty} \left( {\sum\limits_{k = 1}^n {\frac{1}{k}}  - \ln n} \right)\]

E mais abaixo, nessa mesma página apresento (e demonstro) a fórmula produto de Weierstrass

\[
\frac{1}{{\Gamma \left( x \right)}} = xe^{  \gamma x} \prod\limits_{k = 1}^\infty  {\left( {1 + \frac{x}{k}} \right)e^{ - \frac{x}{k}} }
\]

Como função auxiliar vou introduzir a função digama, $\psi$ , que é a derivada logarítmica da Gama, isto é

\[
\psi (x): = \left( {\ln \Gamma \left( x \right)} \right)^\prime   = \frac{\Gamma '\left( x \right)}{\Gamma \left( x \right)}
\]

Da fórmula produto de Weierstrass, (aplicando logaritmos a ambos os membros) é imediato que
\[
 - \ln \Gamma \left( x \right) = \ln x +\gamma x + \sum\limits_{k = 1}^{ + \infty } {\left[ {\ln \left( {\frac{{k + x}}{k}} \right) - \frac{x}{k}} \right]}
\]

Derivando ambos os membros temos
\[ - \psi \left( x \right) = \frac{1}{x} + \gamma  + \sum\limits_{k = 1}^{ + \infty } \left[ \frac{1}{k + x}- \frac{1}{k} \right] \]
Para $x=1$ temos \[ -\frac{\Gamma '\left( 1 \right)}{\Gamma \left( 1 \right)} = 1 + \gamma + \sum\limits_{k = 1}^{ + \infty }\left[ \frac{1}{k + 1} - \frac{1}{k} \right] \]

Como a série do lado direito é uma série de Mengoli que converge para $-1$ e $\Gamma(1)=0!=1$

Sai 

\[-\Gamma'(1)=\gamma\]
$\blacksquare$


PS: A Wikipedia apresenta outros resultados curiosos... :)

24/11/2019

Uma dedução das fórmulas do movimento rectilíneo uniformemente variado

Está na moda a (errada) filosofia de que as demonstrações só interessam aos matemáticos. Sem provas pregam-se dogmas, e não ciência. Biologia, Física, Geologia, Matemática, Química são ciências e não religiões. Uma formação científica decente deve ser capaz de dar demonstrações aos alunos, sem se tornar maçadora e aborrecida.

Hoje vou partir da definição de "movimento rectilíneo uniformemente variado" e deduzir as equações da velocidade e das posições, sem recorrer explícitamente ao cálculo integral.
[Esta prova ocorreu-me ontem num esclarecimento de dúvidas de Física de 11º... porque a vi quando eu estava no meu ensino secundário]

movimento rectilíneo uniformemente variado (m.r.u.v.) é um movimento rectilíneo em que a aceleração é constante e tem um valor $a$.

Se é constante, a aceleração média é também constante e igual ao mesmo valor $a$, ou seja
\[\frac{\Delta v}{\Delta t} =a \]
Isso significa que se o objecto inicia o movimento no instante $t=0$ com velocidade $v_0$, noutro instante $t$ terá velocidade $v$, cuja fórmula pode ser facilmente deduzida:
\begin{eqnarray*} {\frac{\Delta v}{\Delta t} =a}{\Leftrightarrow}{\frac{v-v_0}{t-0} =a}\\ {}{\Leftrightarrow}{v-v_0=at}\\ {}{\Leftrightarrow}{v=v_0+at} \end{eqnarray*}

Portanto a equação das velocidades é \[v=v_0+at\]
Num gráfico velocidade-tempo, esta velocidade é uma função afim, ou seja, tem o gráfico de uma recta.



A área entre o gráfico da velocidade e o eixo dos "$t$" dá-nos a variação de posição $\Delta x$ (porquê?).

E assim, para deduzirmos a equação do movimento basta recordar e aplicar a fórmula da área do trapézio:
\[A= \frac{B+b}{2}\times h\]
Onde $B$ e $b$ são as bases e $h$ a altura do trapézio.
Portanto \[\Delta x=\frac{(v_0+at)+v_0}{2}\times t=v_0t+\frac{1}{2}at^2\] Assumindo que no instante $t$ o objecto está na posição $x$ e no instante $t=0$ estava na posição $x_0$, temos \[x-x_0=v_0t+\frac{1}{2}at^2\]
Ou seja \[x=x_0+v_0t+\frac{1}{2}at^2\]
Que é a equação do movimento do objecto.

03/08/2019

De uma derivada de ordem k à binomial negativa.

Ontem à noite, estava eu sentado, num café, a resolver alguns exercícios de Probabilidades e Estatística. Precisei de umas fórmulas para a binomial negativa. Tinha as fórmulas nos meus apontamentos, mas não tinha a dedução. Já estava meio farto de cálculos "mecânicos".
Estava sem bateria no telemóvel, portanto, estava sem acesso à Internet.
Isto foi o que me ocorreu (isto são só cálculos, mas parece que sou mais produtivo num café do que em casa ou no trabalho):
Sabe-se que se $0<|x|<1$ então \[ \sum\limits_{n = 0}^\infty {x^n } = \frac{1}{{1 - x}} \] Derivando esta expressão termo a termo temos: \[ \sum\limits_{n = 0}^\infty {nx^{n-1} } = \frac{1}{\left(1 - x\right)^2} \] Note-se que o primeiro termo da série é zero , portanto esta igualdade pode ser reescrita \[ \sum\limits_{n = 1}^\infty {nx^{n-1} } = \frac{1}{\left(1 - x\right)^2} \] Voltando a derivar cada um dos membros \[ \sum\limits_{n = 1}^\infty {n(n-1)x^{n-2} } = \frac{2}{\left(1 - x\right)^3} \] E mais uma vez \[ \sum\limits_{n = 2}^\infty {n(n-1)x^{n-2} } = \frac{2}{\left(1 - x\right)^3} \] Continuando a derivar obtemos \[ \color{red}\sum\limits_{n = k}^\infty {n(n-1)\cdots(n-k+1)x^{n-k} } = \frac{k!}{\left(1 - x\right)^{k+1}} \] (Fórmula que pode ser confirmada pelo método de indução)
Onde $n(n-1)\cdots(n-k+1)={}^{n}A_k$ é a conhecida fórmula para arranjos de $n$ $k$ a $k$
Dividindo ambas as expressões por $k!$ obtém-se \[ \color{blue}\sum\limits_{n = k}^\infty {\left(\begin{array}{l} {n}\\ {k} \end{array} \right)x^{n-k} } = \frac{1}{\left(1 - x\right)^{k+1}} \] Fazendo as mudanças de variáveis $N=n+1$ e $K=k+1$, esta igualdade converte-se em \[ \color{green}\sum\limits_{N = K}^\infty {\left(\begin{array}{l} {N-1}\\ {K-1} \end{array} \right)x^{N-K} } = \frac{1}{\left(1 - x\right)^{K}} \]

Aplicação à binomial negativa

Diz-se que uma variável aleatória $X$="número de provas de Bernoulli a realizar até se obterem $k$ sucessos" tem distribuição binomial negativa. A função massa de probabilidade desta distribuição é dada por: \[ f_X(x)=P(X=x)= \left\{ {\begin{array}{l} {\left( {\begin{array}{l} {x - 1} \\ {k - 1} \end{array}} \right)p^k (1 - x)^{x - k}\text{ } x = k,k + 1,k + 2...} \\ {0,\text{caso contrário}} \end{array}} \right. \] Assim sendo \begin{eqnarray*} {E(X)}&{ = }&{\sum\limits_{x=k}^\infty {x f_X(x)}}\\ {}&{=}&{\sum\limits_{x = k}^\infty {x\left( {\begin{array}{l} {x - 1} \\ {k - 1} \end{array}} \right)p^k (1 - p)^{x - k} }}\\ {}&{ = }&{\frac{{p^k }}{{\left( {k - 1} \right)!}}\sum\limits_{x = k}^\infty {\frac{{x!}}{{\left( {x - k} \right)!}}(1 - p)^{x - k} }}\\ {}&{ = } & { \frac{{p^k }}{{\left( {k - 1} \right)!}}{\color{red}\sum\limits_{x = k}^\infty {^x A_k (1 - x)^{x - k} }}}\\ {}&{ = }& {\frac{{p^k }}{{\left( {k - 1} \right)!}}{\color{red}\frac{{k!}}{{\left( {1 - \left( {1 - p} \right)} \right)^{k + 1} }}}}\\ {}&{ = }& {\frac{k}{p}} \end{eqnarray*} A função geradora de momentos é \begin{eqnarray*} {M_X (t)}&{ = }&{ E\left( {e^{tX} } \right) }\\ { }&{ = }&{\sum\limits_{x = k}^\infty {e^{tx} f_X(x)}}\\ { }&{ = }&{\sum\limits_{x = k}^\infty {e^{tx} \left( {\begin{array}{l} {x - 1} \\ {k - 1} \end{array}} \right)p^k (1 - p)^{x - k} } }\\ {}&{ = }&{\sum\limits_{x = k}^\infty {e^{tx - tk} \left( {\begin{array}{l} {x - 1} \\ {k - 1} \end{array}} \right)e^{tk} p^k (1 - p)^{x - k} }}\\ {}&{ = } &{\sum\limits_{x = k}^\infty {\left( {\begin{array}{l} {x - 1} \\ {k - 1} \end{array}} \right)\left( {e^t p} \right)^k \left( {e^t \left( {1 - p} \right)} \right)^{x - k} } } \\ {}&{ = } &{\left( {e^t p} \right)^k {\color{green}\sum\limits_{x = k}^\infty {\left( {\begin{array}{l} {x - 1} \\ {k - 1} \end{array}} \right)\left( {e^t \left( {1 - p} \right)} \right)^{x - k} }}}\\ {}&{ = }&{\left( {e^t p} \right)^k {\color{green}\frac{1}{{\left( {1 - e^t \left( {1 - p} \right)} \right)^k }}}}\\ {}&{ = }&{\left( {\frac{{e^t p}}{{1 - e^t \left( {1 - p} \right)}}} \right)^k} \end{eqnarray*} Com esta função pode-se calcular \[ E(X^2 ) = \lim_\limits{t\to 0} \frac{{d^2 }}{{dt^2 }}\left( {\frac{{e^t p}}{{1 - e^t \left( {1 - p} \right)}}} \right)^k = \frac{{k\left( {k - p + 1} \right)}}{{p^2 }} \]

\begin{eqnarray*} {\frac{d}{{dt}}\left( {M_{X(t)} } \right)}&{ = }&{\frac{d}{{dt}}\left( {\left( {\frac{{e^t \cdot p}}{{1 - e^t \cdot \left( {1 - p} \right)}}} \right)^k } \right)}\\ {}&{ = }&{k\left( {\frac{{e^t \cdot p}}{{1 - e^t \cdot \left( {1 - p} \right)}}} \right)^{k - 1} \frac{d}{{dt}}\left( {\frac{{e^t \cdot p}}{{1 - e^t \cdot \left( {1 - p} \right)}}} \right)} \\ {}&{ = }&{ k\left( {\frac{{e^t \cdot p}}{{1 - e^t \cdot \left( {1 - p} \right)}}} \right)^{k - 1} \frac{{e^t \cdot p\left( {1 - e^t \cdot \left( {1 - p} \right)} \right) + e^t \cdot pe^t \cdot \left( {1 - p} \right)}}{{\left( {1 - e^t \cdot \left( {1 - p} \right)} \right)^2 }}} \\ {}&{ = }&{k\left( {\frac{{e^t \cdot p}}{{1 - e^t \cdot \left( {1 - p} \right)}}} \right)^k \frac{1}{{1 - e^t \cdot \left( {1 - p} \right)}}}\\ {}&{ = }&{\frac{{k \cdot e^{kt} \cdot p^k }}{{\left( {1 - e^t \cdot \left( {1 - p} \right)} \right)^{k + 1} }}} \end{eqnarray*} Passemos à segunda derivada: \begin{eqnarray*} {\frac{{d^2 }}{{dt^2 }}\left( {M_{X(t)} } \right)}&{ = }&{\frac{d}{{dt}}\left( {\frac{d}{{dt}}\left( {M_{X(t)} } \right)} \right)}\\ {}&{ = }&{ \frac{d}{{dt}}\left( {\frac{{k \cdot e^{kt} \cdot p^k }}{{\left( {1 - e^t \cdot \left( {1 - p} \right)} \right)^{k + 1} }}} \right)}\\ {}&{ = }&{\frac{{k^2 \cdot e^{kt} \cdot p^k \left( {1 - e^t \cdot \left( {1 - p} \right)} \right)^{k + 1} - k \cdot e^{kt} \cdot p^k \left( {k + 1} \right)\left( {1 - e^t \cdot \left( {1 - p} \right)} \right)^k \left[ { - e^t \cdot \left( {1 - p} \right)} \right]}}{{\left( {1 - e^t \cdot \left( {1 - p} \right)} \right)^{2k + 2} }}}\\ {}&{=}&{\frac{{ke^{kt} p^k \left[ {k\left( {1 - e^t \cdot \left( {1 - p} \right)} \right) + \left( {k + 1} \right)\left( {1 - p} \right)e^t } \right]}}{{\left( {1 - e^t \cdot \left( {1 - p} \right)} \right)^{k + 2} }}} \end{eqnarray*} Finalmente: \begin{eqnarray*} {E(X^2 )}&{ = }&{\lim_\limits{t\to 0} \frac{d^2 }{dt^2 }\left( M_{X(t)} \right)}\\ {}&{=}&{\frac{{kp^k \left[ {k\left( {1 - \left( {1 - p} \right)} \right) + \left( {k + 1} \right)\left( {1 - p} \right)} \right]}}{{\left( {1 - \left( {1 - p} \right)} \right)^{k + 2} }}}\\ {}&{=}&{\frac{{kp^k \left( {kp + k - kp + 1 - p} \right)}}{{p^{k + 2} }}}\\ {}&{ = }&{\frac{{k\left( {k - p + 1} \right)}}{{p^2 }}} \end{eqnarray*}
E daqui \[ Var(X) = E\left( {X^2 } \right) - E\left( X \right)^2 = \frac{{k\left( {k - p + 1} \right)}}{{p^2 }} - \frac{{k^2 }}{{p^2 }} = \frac{{k\left( {1 - p} \right)}}{{p^2 }} \]
PS:
  • Eu nunca tinha feito isto antes. Em caso de gralhas ou erros, eu vou acabar por corrigir, mas podem contactar-me.
  • Obviamente, existem outras formas de fazer algumas destas coisas... eu sei onde pode ver algumas. Por exemplo, posso sugerir o capítulo 4 da primeira edição do livro Introdução à Probabilidade e à Estatística , de Dinis Pestana e Sílvio Velosa — estou a sugerir a primeira edição porque no dia em que escrevo isto é a que está à minha frente!

20/07/2019

O operador Laplaciano em coordenadas polares e em coordenadas esféricas

O operador Laplaciano apareceu-me muitas vezes em problemas com equações diferenciais com derivadas parciais, em problemas de electromagnetismo, e mais recentemente em problemas de mecânica quântica. Muitas vezes é necessário fazer uma mudança de variáveis, para coordenadas polares, esféricas, cilíndricas...
Normalmente, as fórmulas são dadas, sem qualquer dedução. Não porque a dedução em si seja difícil, mas porque os cálculos em si podem ser longos e não trazem nada de novo. O problema deste ponto de vista é que há quem nunca tenha visto nem feito uma dedução!
É um mero exercício de cálculo e de aplicação de principalmente da regra da derivação do produto e da regra da cadeia, para quem quiser fazer.
...E que proponho que se faça! Clicando nos botões podem ver a minha solução e a minha resolução. Sugiro que tente fazê-la primeiro!

O Laplaciano em coordenadas polares

Seja $f:D \subseteq \R^2 \to \R$ uma função real de variável vectorial, ou, como vai ser designação comum neste blog, um campo escalar.
Suponhamos que a função é de classe $C^2$ em $D$.
O Laplaciano de $f$ é a divergência do gradiente de $f$. \[ \nabla ^2 f = \nabla \cdot \nabla f = \dfrc{{\partial ^2 f}}{{\partial x{}^2}} + \dfrc{{\partial ^2 f}}{{\partial y{}^2}} \] Considere-se a nova função que se obtém fazendo a mudança de variáveis \[ \left\{ {\begin{array}{l} {x = r\cos \theta } \\ {y = r\sen \theta } \end{array}} \right. \] para $r>0$ e $\theta\in [0,2\pi[ $ por forma a que, a nova definição faça sentido.
Por abuso de linguagem, continuemos a designar a nova função por $f$.
Determine uma nova fórmula para $\nabla ^2 f$ em função das novas variáveis $r$ e $\theta$.
\[\nabla ^2 f=\dfrc{{\partial ^2 f}}{{\partial r^2 }} + \dfrc{1}{r}\dfrc{\partial f}{\partial r} + \dfrc{1}{{r^2 }}\dfrc{{\partial ^2 f}}{{\partial \theta ^2 }}\]
(Carlos Paulo A. de Freitas; 19/07/2019)
\[ \left\{ {\begin{array}{l} {x = r\cos \theta } \\ {y = r\sen \theta } \end{array}} \right. \Rightarrow \left\{ {\begin{array}{l} {r^2 = x^2 + y^2 } \\ \cos \theta = \dfrc{x}{r} \\ \sen \theta = \dfrc{y}{r} \\ \end{array}} \right. \Rightarrow \left\{ {\begin{array}{l} {2r\dfrc{\partial r}{\partial x} = 2x} \\ 2r\dfrc{\partial r}{\partial y} = 2y \\ - \sen \theta \dfrc{{\partial \theta }}{\partial x} = \dfrc{{r - x\dfrc{\partial r}{\partial x}}}{{r^2 }} \\ \cos \theta \dfrc{{\partial \theta }}{\partial y} = \dfrc{{r - y\dfrc{\partial r}{\partial y}}}{{r^2 }} \end{array}} \right. \Rightarrow \left\{ {\begin{array}{l} {\dfrc{\partial r}{\partial x} = \dfrc{x}{r} = \cos \theta } \\ \dfrc{\partial r}{\partial y} = \dfrc{y}{r} = \sen \theta \\ \dfrc{\partial \theta }{\partial x} = \dfrc{{r - r\cos ^2 \theta }}{{ - r^2 \sen \theta }} = \dfrc{{1 - \cos ^2 \theta }}{{ - r\sen \theta }} = - \dfrc{\sen \theta }{r} \\ \dfrc{\partial \theta }{\partial y} = \dfrc{r - r\sen ^2 \theta }{r^2 \cos \theta } = \dfrc{\cos \theta }{r} \end{array}} \right. \] Portanto:
${\color{red}\dfrc{\partial r}{\partial x}=\cos\theta}$ ; ${\color{green}\dfrc{\partial r}{\partial y}=\sen\theta}$ ; $\color{blue} \dfrc{\partial \theta }{\partial x}=- \dfrc{\sen \theta }{r}$ e $\color{brown} \dfrc{\partial \theta }{\partial y} =\dfrc{\cos \theta }{r}$
Estes resultados agora usam-se neste desenvolvimento: \begin{eqnarray*} { \nabla ^2 f }&{=}&{\dfrc{\partial ^2 f}{\partial x^2} + \dfrc{\partial ^2 f}{\partial y^2}}\\ {}&{=}&{\dfrc{\partial }{\partial x}\left( \dfrc{\partial f}{\partial x} \right) + \dfrc{\partial }{\partial y}\left( {\dfrc{\partial f}{\partial y}} \right)}\\ {}&{=}&{\dfrc{\partial }{\partial x}\underbrace{\left(\dfrc{\partial f}{\partial r}{\color{red}\dfrc{\partial r}{\partial x}} + \dfrc{\partial f}{\partial \theta}{\color{blue} \dfrc{\partial \theta }{\partial x}}\right)}_{\text{regra da cadeia}} + \dfrc{\partial }{\partial y}\underbrace{\left(\dfrc{\partial f}{\partial r}{\color{green}\dfrc{\partial r}{\partial y}} + \dfrc{\partial f}{\partial \theta }\color{brown} \dfrc{\partial \theta }{\partial y}\right)}_{\text{regra da cadeia}} }\\ {}&{=}&{\dfrc{\partial }{\partial x}\left( \dfrc{\partial f}{\partial r}\cos \theta - \dfrc{\partial f}{\partial \theta }\dfrc{\sen \theta }{r} \right) + \dfrc{\partial }{\partial y}\left( \dfrc{\partial f}{\partial r}\sen \theta + \dfrc{\partial f}{\partial \theta}\dfrc{\cos \theta }{r} \right)}\\ {}&{=}&{\dfrc{\partial }{\partial r}\left( {\dfrc{\partial f}{\partial r}\cos \theta - \dfrc{\partial f}{\partial \theta }\dfrc{\sen \theta }{r}} \right){\color{red}\dfrc{\partial r}{\partial x}} + \dfrc{\partial }{\partial \theta }\left( {\dfrc{\partial f}{\partial r}\cos \theta - \dfrc{\partial f}{\partial \theta }\dfrc{\sen \theta }{r}} \right){\color{blue} \dfrc{\partial \theta }{\partial x}}}\\ {}&{ }&{+ \dfrc{\partial }{\partial r}\left(\dfrc{\partial f}{\partial r}\sin \theta + \dfrc{\partial f}{\partial \theta }\dfrc{\cos \theta }{r} \right){\color{green}\dfrc{\partial r}{\partial y}} + \dfrc{\partial }{\partial \theta }\left( {\dfrc{\partial f}{\partial r}\sin \theta + \dfrc{\partial f}{\partial \theta }}\dfrc{\cos \theta }{r} \right){\color{brown} \dfrc{\partial \theta }{\partial y}} }\\ {}&{}&{\text{(regra da cadeia)}}\\ {}&{=}&{\left( \dfrc{\partial ^2 f}{\partial r^2 }\cos \theta + \dfrc{\sen \theta}{r^2 }\dfrc{\partial f}{\partial \theta} - \dfrc{\sen \theta}{r}\dfrc{\partial ^2 f}{\partial r\partial \theta } \right){\color{red}\dfrc{\partial r}{\partial x}} + \left( {\dfrc{\partial ^2 f}{\partial \theta \partial r}\cos \theta - \dfrc{\partial f}{\partial r}\sin \theta - \dfrc{\cos \theta}{r}\dfrc{\partial f}{\partial \theta} - \dfrc{\sen \theta}{r}\dfrc{\partial ^2 f}{\partial \theta ^2 }} \right){\color{blue} \dfrc{\partial \theta }{\partial x}} }\\ {}&{}&{+ \left( \dfrc{\partial ^2 f}{\partial r^2 }\sin \theta - \dfrc{\cos \theta}{r^2 }\dfrc{\partial f}{\partial \theta} + \dfrc{\cos \theta}{r}\dfrc{\partial ^2 f}{\partial r\partial \theta } \right){\color{green}\dfrc{\partial r}{\partial y}} + \left( \dfrc{\partial ^2 f}{\partial \theta \partial r}\sin \theta + \dfrc{\partial f}{\partial r}\cos \theta - \dfrc{\sen \theta}{r}\dfrc{\partial f}{\partial \theta} + \dfrc{\cos \theta}{r}\dfrc{\partial ^2 f}{\partial \theta ^2 } \right){\color{brown}\dfrc{\partial \theta}{\partial y}} }\\ {}&{=}&{\dfrc{\partial ^2 f}{\partial r^2 }\cos ^2 \theta + \dfrc{\sin \theta \cos \theta }{r^2 }\dfrc{\partial f}{\partial \theta} - \dfrc{\sen \theta \cos \theta }{r}\dfrc{\partial ^2 f}{\partial r\partial \theta } - \dfrc{\sen \theta \cos \theta }{r}\dfrc{\partial ^2 f}{\partial \theta \partial r} + \dfrc{\partial f}{\partial r}\dfrc{\sin ^2 \theta }{r} + \dfrc{\sin \theta \cos \theta }{r^2 }\dfrc{\partial f}{\partial \theta} + \dfrc{\sin ^2 \theta }{r^2 }\dfrc{\partial ^2 f}{\partial \theta ^2 } }\\ {}&{ }&{+ \dfrc{\partial ^2 f}{\partial r^2 }\sin ^2 \theta - \dfrc{\sin \theta \cos \theta }{r^2 }\dfrc{\partial f}{\partial \theta} + \dfrc{\sen \theta \cos \theta }{r}\dfrc{\partial ^2 f}{\partial r\partial \theta } + \dfrc{\sin \theta \cos \theta }{r}\dfrc{\partial ^2 f}{\partial \theta \partial r} + \dfrc{\partial f}{\partial r}\dfrc{\cos ^2 \theta }{r} - \dfrc{\sen \theta \cos \theta }{r^2 }\dfrc{\partial f}{\partial \theta} + \dfrc{\cos ^2 \theta }{r^2 }\dfrc{\partial ^2 f}{\partial \theta ^2 } }\\ {}&{=}&{\dfrc{\partial ^2 f}{\partial r^2 } + \dfrc{1}{r}\dfrc{\partial f}{\partial r} + \dfrc{1}{r^2 }\dfrc{\partial ^2 f}{\partial \theta ^2 }} \end{eqnarray*}

O Laplaciano em coordenadas esféricas

Seja $f:D \subseteq \R^3 \to \R$ um campo escalar.
Suponhamos que a função é de classe $C^2$ em $D$.
O Laplaciano de $f$ é a divergência do gradiente de $f$. \[ \nabla ^2 f = \nabla \cdot \nabla f = \dfrc{\partial ^2 f}{\partial x^2} + \dfrc{\partial ^2 f}{\partial y^2}+ \dfrc{\partial ^2 f}{\partial z^2} \] Considere-se a nova função que se obtém fazendo a mudança de variáveis \[ \left\{ {\begin{array}{l} {x = r\sen \theta \cos \phi} \\ {y = r\sen \theta \sen \phi} \\ {z = r\cos \theta} \end{array}} \right. \] para $r>0$, $\theta\in [0,\pi[ $ e $\phi\in [0,2\pi[ $ por forma a que, a nova definição faça sentido.
Por abuso de linguagem, continuemos a designar a nova função por $f$.
Mostre que \[ \nabla ^2 f = \frac{1}{r^2}\left[\frac{\partial}{\partial r} \left( {r^2}\dfrc{\partial f}{\partial r} \right) + \frac{1}{ \sen \theta }\frac{\partial }{{\partial \theta }}\left( {\sen \theta \frac{{\partial f}}{{\partial \theta }}} \right) + \frac{1}{{ \sen ^2 \theta }}\frac{{\partial ^2 f}}{{\partial \phi ^2 }}\right] \]
Nota: Em breve deixo a minha dedução, análoga à dedução da fórmula em coordenadas polares.

01/07/2019

Uma equação com problemas (I)

Há uns meses chegou-me um explicando com o seguinte problema.

Seja $k\in \R \backslash \{0\} $. Calcule um valor de $k$ de modo que:
\[
\int\limits_k^\pi  {\frac{1}{{\left( {2 + \sin ^2 \left( {2x} \right)} \right)\cos ^{ - 1} \left( {2x} \right)}}dx =  - \frac{{\sqrt 2 }}{2}\pi }
\]

Ao resolver a equação percebi que a equação era impossível!
Abaixo (botão) deixo a minha resolução feita nesse dia.


\begin{eqnarray*} {\int\limits_k^\pi {\frac{1}{{\left( {2 + \sin ^2 \left( {2x} \right)} \right)\cos ^{ - 1} \left( {2x} \right)}}dx}}& {=} &{ - \frac{{\sqrt 2 }}{2}\pi }\\ {\Leftrightarrow \int\limits_k^\pi {\frac{{\cos \left( {2x} \right)}}{{2 + \sin ^2 \left( {2x} \right)}}dx}}&{ = }&{ - \frac{{\sqrt 2 }}{2}\pi }\\ {\Leftrightarrow \frac{1}{2}\int\limits_k^\pi {\frac{{\cos \left( {2x} \right)}}{{1 + \left( {\displaystyle\frac{{\sin \left( {2x} \right)}}{{\sqrt 2 }}} \right)^2 }}dx}} &{=}& { - \frac{{\sqrt 2 }}{2}\pi }\\ {\Leftrightarrow \frac{1}{2}\frac{{\sqrt 2 }}{2}\int\limits_k^\pi {\displaystyle\frac{{\displaystyle\frac{2}{{\sqrt 2 }}\cos \left( {2x} \right)}}{{1 + \left( {\displaystyle\frac{{\sin \left( {2x} \right)}}{{\sqrt 2 }}} \right)^2 }}dx}}&{ = }&{ - \frac{{\sqrt 2 }}{2}\pi } \\ {\Leftrightarrow \int\limits_k^\pi {\frac{{\displaystyle\frac{2}{{\sqrt 2 }}\cos \left( {2x} \right)}}{{1 + \left( {\displaystyle\frac{{\sin \left( {2x} \right)}}{{\sqrt 2 }}} \right)^2 }}dx}}&{ = }& { - 2\pi } \\ {\Leftrightarrow - \arctg\left( {\frac{{\sin \left( {2k} \right)}}{{\sqrt 2 }}} \right)}&{ = }&{ - 2\pi } \\ {\Leftrightarrow \arctg\left( {\frac{{\sin \left( {2k} \right)}}{{\sqrt 2 }}} \right)}&{ = }&{ 2\pi } \\ \end{eqnarray*} Uma vez que o contradomínio da função arcotangente é $\left] { -\displaystyle \frac{\pi }{2},\displaystyle\frac{\pi }{2}} \right[$, a equação obtida é impossível.
Quero que compreendam que mesmo deslocando o contradomínio do arcotangente por um múltiplo inteiro de $\pi$, a equação original continua a ser impossível, visto que essa constante "desapareceria" no decorrer dos cálculos (nomeadamente depois de aplicação da regra de Barrow).


Escrever \[{\frac{{\sin \left( {2k} \right)}}{{\sqrt 2 }}}=\tg (2\pi)\] É um erro gravíssimo!!! (e que demonstra sério e grave desconhecimento do que se anda a fazer)!

Mas, para que não fiquem dúvidas na cabeça mais teimosa, vou provar a impossibilidade daquela equação de outra forma.
Considere-se a função: \[ F(k) = \int\limits_k^\pi {\frac{1}{{\left( {2 + \sin ^2 \left( {2x} \right)} \right)\cos ^{ - 1} \left( {2x} \right)}}dx} = - \frac{{\sqrt 2 }}{4}\arctg\left( {\frac{{\sin \left( {2k} \right)}}{{\sqrt 2 }}} \right) \] Ora, como \[- 1 \le \sin \left( {2k} \right) \le 1 \] então \[\frac{{ - 1}}{{\sqrt 2 }} \le \frac{{\sin \left( {2k} \right)}}{{\sqrt 2 }} \le \frac{1}{{\sqrt 2 }}\] Então isto implica que \[{{ - 1}} \leq \frac{{\sin \left( {2k} \right)}}{{\sqrt 2 }} \leq {1}\] \[\Leftrightarrow - \frac{\pi }{4} \le \arctg\left( {\frac{{\sin \left( {2k} \right)}}{{\sqrt 2 }}} \right) \le \frac{\pi }{4}\] \[ \Leftrightarrow - \frac{{\pi \sqrt 2 }}{{16}} \le - \frac{{\sqrt 2 }}{4}\arctg\left( {\frac{{\sin \left( {2k} \right)}}{{\sqrt 2 }}} \right) \le \frac{{\pi \sqrt 2 }}{{16}}\] \[ \Leftrightarrow - \frac{{\pi \sqrt 2 }}{{16}} \le F(k) \le \frac{{\pi \sqrt 2 }}{{16}}\] Esta última condição prova que $-\displaystyle\frac{{\sqrt 2 }}{2}\pi$ está fora do contradomínio de $F$, portanto a equação original é impossível.

Numa última nota:
  • Não percebi porque se exige no enunciado que $k\neq 0$.   Faz-me suspeitar que há algo de errado com o enunciado.
  • Eu 'verifiquei' numericamente as minhas afirmações antes de me dar ao trabalho de escrever isto... podia ter erros nos cálculos.
  • Como sempre, se encontrarem gralhas ou incorrecções, podem enviar-me para cpaulof at gmail dot com

Actualizações:
  • 03/07/2019: $\cos^{-1}$ deve ser interpretado como a função secante, e não como a função arco-cosseno, graças aos valores dos limites de integração.
  • 03/07/2019: em vez de $k\in\R\backslash \{0\}$ penso que $k$ deve pertencer a um subconjunto de $\R\backslash \left\{x=\displaystyle\frac{\pi}{4}+\frac{n\pi}{2}, n\in \Z\right\}$, mas não sei. O enunciado não é meu.


    \begin{eqnarray*} {D }&{=}&{ \left\{ {x \in \R:\cos \left( {2x} \right) \ne 0} \right\}}\\ {}&{ = }&{\left\{ {x \in \R:2x \ne \frac{\pi }{2} + n\pi ;n \in \Z} \right\}}\\ {}&{ = }&{ \left\{ {x \in \R:x \ne \frac{\pi }{4} + \frac{{n\pi }}{2};n \in \Z} \right\}} \end{eqnarray*}
  • 09/11/2021: Encontrei uma gralha, mas corrigi... ninguém se deu ao trabalho de me avisar!

07/06/2019

Uma parametrização para uma faixa de Möbius

Recentemente, depois de ver o filme "Avengers: Endgame", onde a solução de Tony Stark para um problema físico (não vou dar mais spoilers do que o necessário, se quiserem, vão lá ver o filme) passava por uma faixa de Möbius, andei a fazer umas experiências com a faixa de Möbius.
Não me apeteceu utilizar a Wikipedia... apeteceu-me usar o cérebro.

(Algo que até faço quando uso uma calculadora...quem diria, não é?)

Passo 0:

O que é uma faixa/fita de Möbius?

De forma muito simplista, neste post, a minha faixa de Möbius vai ser a superfície descrita por um segmento de recta que percorre uma circunferência enquanto roda meia volta sobre o seu centro.
Vejam a animação seguinte que percebem melhor,
  

Se calhar fica melhor se eu mostrar a superfície a ser desenhada:

Portanto, é boa ideia começar por parametrizar um segmento de recta que dá meia volta!

Passo 1: No plano $xOy$ parametrizar um segmento de recta, centrado na origem, que dá meia volta, em torno da origem.



Dados dois pontos distintos do plano ou do espaço, $A$ e $B$, o segmento $[BA]$ pode ser parametrizado por .
\[X=tA+(1-t)B\] com $t \in [0,1]$

Portanto, se, para um ângulo qualquer de amplitude $\alpha$ considerarmos os pontos da circunferência trigonométrica
\[A=(\cos \alpha,\sen \alpha) \] e \[B=(\cos (\alpha + \pi), \sen (\alpha + \pi)) \]. que são simétricos relativamente à origem $O ( 0,0 )$

O segmento é o conjunto dos pontos
\begin{eqnarray*}
{(x,y)}&=&{t\left(\cos \alpha,\sen \alpha)+(1-t)(\cos (\alpha + \pi), \sen (\alpha + \pi)\right)}\\
{}&=&{\left(t\cos \alpha +(1-t)(\cos (\alpha + \pi),(t\sen \alpha +(1-t)(\sen (\alpha + \pi)\right)}
 \end{eqnarray*}
 Para termos a "meia volta" podemos deixar o ângulo $\alpha$ variar entre $0$ e $\pi$.

Mas como a ideia é que o segmento dê meia volta enquanto percorre uma circunferência, então vou substituir o $\alpha$ por um $\displaystyle \frac{\theta}{2}$ com $\theta$ a variar entre $0$ e $2\pi$

Passo 2: Fazer este segmento girar enquanto percorre uma circunferência de raio $R$
No espaço tridimensional $\R^3$ uma circunferência de raio $R$ centrada na origem e contida no plano $z=0$ pode ser parametrizada por \[ (x,y,z)=(R\cos \theta, R \sen \theta, 0 ) \] com \[0\leq\theta <2\pi\]
Para obter o efeito desejado, basta ver que a "semivolta" é dada apenas nas componentes $x$ e $z$, portanto, a equação do percurso do segmento de recta tem de ser:
\begin{eqnarray*}
{ (x,y,z)}&=&{(R\cos \theta, R \sen \theta, 0 ) + \left(t\cos \left(\frac{\theta}{2}\right)+ (1-t)\cos \left(\frac{\theta}{2} + \pi\right),0,t\sen \left(\frac{\theta}{2}\right) + (1-t)\sen \left(\frac{\theta}{2} + \pi\right)\right)}\\
{}&=&{\left(R\cos \theta +t\cos \left(\frac{\theta}{2}\right)+ (1-t)\cos \left(\frac{\theta}{2} + \pi\right), R \sen \theta, t\sen \left(\frac{\theta}{2}\right)+ (1-t)\sen \left(\frac{\theta}{2} + \pi\right) \right)}
\end{eqnarray*}
Passo 3: conclusão.
Então uma parametrização para esta faixa de Möbius é
\[ (x,y,z)=\left(R\cos \theta +t\cos \left(\frac{\theta}{2}\right)+ (1-t)\cos \left(\frac{\theta}{2} + \pi\right), R \sen \theta, t\sen \left(\frac{\theta}{2}\right)+ (1-t)\sen \left(\frac{\theta}{2} + \pi\right) \right)\] Com $0\leq t\leq 1$ e $0\leq \theta < 2\pi $.

A animação que se segue foi gerada com estas equações no software Geogebra


  • Percebeu a dedução? Se sim, qual o comprimento do segmento de recta que está a girar? Como ficaria a parametrização para um segmento de comprimento $L$, centrado na origem?
  • As funções trigonométricas dos ângulos $\left(\displaystyle\frac{\theta}{2} + \pi\right)$ não foram simplificadas de propósito. Se assim o desejar, simplifique a expressão obtida o melhor que conseguir.

03/03/2019

O número de divisores, a soma e o produto dos divisores naturais de um número natural

Recentemente encontrei um problema num dos meus programas de calculadora dos anos 90 a correr numa máquina actual. Na verdade o "problema" deve-se a ter sido escrito numa máquina diferente, e ao fabricante (a CASIO) ter feito algumas modificações nas calculadoras.
Problema facilmente resolúvel. O programa chama-se "números" e uma vez introduzido um número natural dá ao utilizador a decomposição em factores primos, a função $\varphi$ de Euler, o número de divisores, a soma dos divisores e o produto dos divisores, e como bónus até podia mostrar todos os divisores do número.
O programa foi escrito para me servir de apoio numa disciplina de Teoria dos Números, visto que na altura eu estava com um sério problema de saúde e tinha sérios problemas em concentrar-me (aliás, foi nesse ano em que pela primeira vez tive de desistir numa frequência e deixar para exame).

A função $\varphi$ de Euler, dá, para cada natural $n$ o número de números naturais entre 1 e $n-1$ (inclusive) que é coprimo com $n$, ou, por outras palavras, \[ \varphi(n)= \#\left\{m\in \N_1:m<n \land \left(m \text{ e } n \text{ são primos entre si }\right)\right\}\]
Abaixo vou propor um exercício sem indicar as fórmulas para o resolver, e que é rapidamente resolvido por esse programa de calculadora

Exercício:
Considere o número $n=25401600$. Para este número determine:
  • Decomposição de $n$ em factores primos
  • número de divisores de $n$
  • soma dos divisores de $n$
  • produto dos divisores de $n$
  • $\varphi(n)$
  • Os divisores de $n$
  • Decomposição de $25401600$ em factores primos:
    $25401600=2^8\times3^4\times5^2\times7^2$

    $25401600$$2$
    $12700800$$2$
    $6350400$$2$
    $3175200$$2$
    $1587600$$2$
    $793800$$2$
    $396900$$2$
    $198450$$2$
    $99225$$3$
    $33075$$3$
    $11025$$3$
    $3675$$3$
    $1225$$5$
    $245$$5$
    $49$$7$
    $7$$7$
    $1$

  • O número de divisores de $25401600$ é $405$

    número de divisores de $25401600$ = $\tau(25401600)=(8+1)\times(4+1)\times(2+1)\times(2+1)=405$
  • soma dos divisores de $25401600$: $109255377$

    soma dos divisores de $25401600$ = $\displaystyle\frac{2^{8+1}-1}{2-1}\times\displaystyle\frac{3^{4+1}-1}{3-1}\times\displaystyle\frac{5^{2+1}-1}{5-1}\times\displaystyle\frac{7^{2+1}-1}{7-1}=109255377$
  • produto dos divisores de $25401600 \approx 3,050473527\times 10^{1499}$:

    produto dos divisores de $25401600$ = $25401600^{\frac{\tau(25401600)}{2}}=25401600^{\frac{405}{2}}=\\ 3050473527291822531740041345293141334639821433214236708667640373777187466846318630194\\ 9487298892807258697738201087769997689322881899874797172563978299167031514727744821435\\ 3785218826732663217210386436311097076444607289013988610404724784345668111668186460126\\ 9755883592179122020126123180126841834726986839920892479831881375115138978330538550274\\ 8162513638371583118781073185971083062975611933081110856507983196894999341496394996509\\ 3122171965532585683279551649783233634875314042656032015349875338898739359449069030654\\ 4622884131846965196597429175359622576520634929794994536897916292205148207319758296280\\ 8761845614002132058673973259309044737132095162540601973927995102974372145280524186218\\ 4767447352454155282281847316301542497360257699942103612325756704426945835441772896564\\ 4149847241035507859817472009316357318712266669730563110807016171044701603460590155380\\ 6797553649708463435983769227152753101366622137813294565298246991679701906184276978850\\ 6889738588331844184392532814011815755326497291985162421953577246183809913211380900800\\ 4097694439501323737492915590833287001263683936187738812398146247639422730240000000000\\ 0000000000000000000000000000000000000000000000000000000000000000000000000000000000000\\ 0000000000000000000000000000000000000000000000000000000000000000000000000000000000000\\ 0000000000000000000000000000000000000000000000000000000000000000000000000000000000000\\ 0000000000000000000000000000000000000000000000000000000000000000000000000000000000000\\ 0000000000000000000000000000000000000000000000000000000
    \approx 3,050473527\times 10^{1499}$
  • $\varphi(25401600)=5806080$

    $\varphi(25401600)=2^{8-1}(2-1)\times3^{4-1}(3-1)\times5^{2-1}(5-1)\times7^{2-1}(7-1)=5806080$
  • $D_{25401600}=\left\{\right.$ 1, 2, 3, 4, 5, 6, 7, 8, 9, 10, 12, 14, 15, 16, 18, 20, 21, 24, 25, 27, 28, 30, 32, 35, 36, 40, 42, 45, 48, 49, 50, 54, 56, 60, 63, 64, 70, 72, 75, 80, 81, 84, 90, 96, 98, 100, 105, 108, 112, 120, 126, 128, 135, 140, 144, 147, 150, 160, 162, 168, 175, 180, 189, 192, 196, 200, 210, 216, 224, 225, 240, 245, 252, 256, 270, 280, 288, 294, 300, 315, 320,324, 336, 350, 360, 378, 384, 392, 400, 405, 420, 432, 441, 448, 450, 480, 490, 504, 525, 540, 560, 567, 576, 588, 600, 630, 640, 648, 672, 675, 700, 720, 735, 756, 768, 784, 800, 810, 840, 864, 882, 896, 900, 945, 960, 980, 1008, 1050, 1080, 1120, 1134, 1152, 1176, 1200, 1225, 1260, 1280, 1296, 1323, 1344, 1350, 1400, 1440, 1470, 1512, 1568, 1575, 1600, 1620, 1680, 1728, 1764,1792, 1800, 1890, 1920, 1960, 2016, 2025, 2100, 2160, 2205, 2240, 2268, 2304, 2352, 2400, 2450, 2520, 2592, 2646, 2688, 2700,2800, 2835, 2880, 2940, 3024, 3136, 3150, 3200, 3240, 3360, 3456, 3528, 3600, 3675, 3780, 3840, 3920, 3969, 4032, 4050, 4200, 4320, 4410, 4480, 4536, 4704, 4725, 4800, 4900, 5040, 5184, 5292, 5376, 5400, 5600, 5670, 5760, 5880, 6048, 6272, 6300, 6400,6480, 6615, 6720, 6912, 7056, 7200, 7350, 7560, 7840, 7938, 8064, 8100, 8400, 8640, 8820, 8960, 9072, 9408, 9450, 9600, 9800,10080, 10368, 10584, 10800, 11025, 11200, 11340, 11520, 11760, 12096, 12544, 12600, 12960, 13230, 13440, 14112, 14175, 14400, 14700, 15120, 15680, 15876, 16128, 16200, 16800, 17280, 17640, 18144, 18816, 18900, 19200, 19600, 19845, 20160, 20736, 21168, 21600, 22050, 22400, 22680, 23520, 24192, 25200, 25920, 26460, 26880, 28224, 28350, 28800, 29400, 30240, 31360, 31752, 32400, 33075, 33600, 34560, 35280, 36288, 37632, 37800, 39200, 39690, 40320, 42336, 43200, 44100, 44800, 45360, 47040, 48384, 50400, 51840, 52920, 56448, 56700, 57600, 58800, 60480, 62720, 63504, 64800, 66150, 67200, 70560, 72576, 75600, 78400, 79380, 80640, 84672, 86400, 88200, 90720, 94080, 99225, 100800, 103680, 105840, 112896, 113400, 117600, 120960, 127008, 129600, 132300, 134400, 141120, 145152, 151200, 156800, 158760, 169344, 172800, 176400, 181440, 188160, 198450, 201600, 211680, 226800, 235200, 241920, 254016, 259200, 264600, 282240, 302400, 313600, 317520, 338688, 352800, 362880, 396900, 403200, 423360, 453600, 470400, 508032, 518400, 529200, 564480, 604800, 635040, 705600, 725760, 793800, 846720, 907200, 940800, 1016064, 1058400, 1209600, 1270080, 1411200, 1587600, 1693440, 1814400, 2116800, 2540160, 2822400, 3175200, 3628800, 4233600, 5080320, 6350400, 8467200, 12700800, 25401600 $\left.\right\}$


Programas de calculadora: (.g1m - Modelos Casio fx-9860GII e fx9750GII; .g3m - Modelos Casio fx-cg10 fx-cg20 e fx-cg50; .8xp - Modelos Texas Instruments TI-84Plus CE e CET, .tns - Modelos Texas Instruments nSpire CX e nSpire CX CAS)

.g1m .g3m .8xp .tns
[Editado a 19-10-2021: Adicionei a versão do programa para TI-84Plus CE]

31/01/2019

Uma mente diferente...

Recentemente, uma explicanda trouxe-me o problema: \[ \left\{ {\begin{array}{l} {u_1 = 3} \\ {u_{n + 1} = u_n + 2n, \text{ se }n > 1} \end{array}} \right. \] Qual o valor de $\displaystyle\sum\limits_{n = 1}^{100} {u_n}$?
Respondi imediatamente que esse problema estava fora do programa da disciplina dela, mas que podia resolvê-lo na boa, e sem calculadora!
Como estava fora de questão recorrer aos meus conhecimentos de equações com diferenças, tive de ser criativo.
\begin{eqnarray*} {u_1}&{=}&{3}\\ {u_2}&{=}&{3+2}\\ {u_3}&{=}&{3+2+4}\\ {u_4}&{=}&{3+2+4+6}\\ {u_5}&{=}&{3+2+4+6+8}\\ {u_6}&{=}&{3+2+4+6+8+10}\\ {...}&{...}&{...}\\ {u_n}&{=}&{3+2\cdot\frac{(n-1)n}{2}=3+(n-1)n} \end{eqnarray*} Justifiquei-lhe o último passo com a fórmula da soma dos primeiros $n$ termos de uma progressão aritmética.
Para o passo sequinte, o cálculo do somatório, tive de ser um pouco mais criativo.
Comecei por escrever \[\sum\limits_{n = 1}^{100} {u_n}=\sum\limits_{n = 1}^{100} {\left(3+(n-1)n\right)}=300+\sum\limits_{n = 1}^{100} {n^2}-5050\] Justifiquei convenientemente os números $300$ e o $5050$, contando a famosa história de Gauss.
Mas para o somatório dos quadrados, não me lembrava da fórmula de cor, embora me apareça regularmente em exercícios de indução. Sabia deduzi-la com equações com diferenças, coisa que eu tinha de evitar porque a explicanda desconhecia.
Mas ao olhar para o papel quadriculado, ocorreu-me a fórmula: \[\sum\limits_{n = 1}^{100} {n^2}=100\times1+99\times 3+98\times 5 + ... + 1 \times (200-1) = \sum\limits_{n = 1}^{100} {(101-n)(2n-1)}\] Consegue percebê-la sem eu partilhar um desenho?


E agora, com este desenho?
Se fizermos $S=\displaystyle\sum\limits_{n = 1}^{100} {n^2}$, a fórmula anterior consegue reescrever-se na forma \[S=-2S+\sum\limits_{n = 1}^{100} {203n}-101\times 100 \] Que nos leva a $3S=1015050$ (número curioso) e portanto $S=338350$.
Logo \[\sum\limits_{n = 1}^{100} {u_n}=300+338350-5050=333600\] Ao que ela respondeu-me: "Percebi, mas não deve ser para resolver assim."
PS: Quando se foi embora, escrevi um programa na calculadora que confirmou a minha solução...